Strengthening Bertrand's postulate using the prime number theorem [duplicate]












0












$begingroup$



This question already has an answer here:




  • How far to nearest/next prime?

    1 answer




In its page on Bertrand's postulate Wikipedia says




It follows from the prime number theorem that for any real
$varepsilon >0$ there is an $n_0 > 0$ such that for all $n > n_0$
there is a prime $p $ such that $n < p < ( 1 + ε )n$ .




I need to quote this result for $varepsilon = 1/2$. I'd like a reference more formal than Wikipedia.










share|cite|improve this question











$endgroup$



marked as duplicate by hardmath, user91500, Lord_Farin, Cesareo, Jyrki Lahtonen Jan 17 at 15:15


This question has been asked before and already has an answer. If those answers do not fully address your question, please ask a new question.














  • 1




    $begingroup$
    Why don't you cite the reference that very source that Wikipedia used: G. H. Hardy and E. M. Wright, An Introduction to the Theory of Numbers, 6th ed., Oxford University Press, 2008, p. 494.
    $endgroup$
    – fleablood
    Jul 16 '18 at 0:00










  • $begingroup$
    @fleablood Of course. I missed that reference. Post as an answer, or I can delete the question as too narrow.
    $endgroup$
    – Ethan Bolker
    Jul 16 '18 at 0:04
















0












$begingroup$



This question already has an answer here:




  • How far to nearest/next prime?

    1 answer




In its page on Bertrand's postulate Wikipedia says




It follows from the prime number theorem that for any real
$varepsilon >0$ there is an $n_0 > 0$ such that for all $n > n_0$
there is a prime $p $ such that $n < p < ( 1 + ε )n$ .




I need to quote this result for $varepsilon = 1/2$. I'd like a reference more formal than Wikipedia.










share|cite|improve this question











$endgroup$



marked as duplicate by hardmath, user91500, Lord_Farin, Cesareo, Jyrki Lahtonen Jan 17 at 15:15


This question has been asked before and already has an answer. If those answers do not fully address your question, please ask a new question.














  • 1




    $begingroup$
    Why don't you cite the reference that very source that Wikipedia used: G. H. Hardy and E. M. Wright, An Introduction to the Theory of Numbers, 6th ed., Oxford University Press, 2008, p. 494.
    $endgroup$
    – fleablood
    Jul 16 '18 at 0:00










  • $begingroup$
    @fleablood Of course. I missed that reference. Post as an answer, or I can delete the question as too narrow.
    $endgroup$
    – Ethan Bolker
    Jul 16 '18 at 0:04














0












0








0





$begingroup$



This question already has an answer here:




  • How far to nearest/next prime?

    1 answer




In its page on Bertrand's postulate Wikipedia says




It follows from the prime number theorem that for any real
$varepsilon >0$ there is an $n_0 > 0$ such that for all $n > n_0$
there is a prime $p $ such that $n < p < ( 1 + ε )n$ .




I need to quote this result for $varepsilon = 1/2$. I'd like a reference more formal than Wikipedia.










share|cite|improve this question











$endgroup$





This question already has an answer here:




  • How far to nearest/next prime?

    1 answer




In its page on Bertrand's postulate Wikipedia says




It follows from the prime number theorem that for any real
$varepsilon >0$ there is an $n_0 > 0$ such that for all $n > n_0$
there is a prime $p $ such that $n < p < ( 1 + ε )n$ .




I need to quote this result for $varepsilon = 1/2$. I'd like a reference more formal than Wikipedia.





This question already has an answer here:




  • How far to nearest/next prime?

    1 answer








reference-request analytic-number-theory






share|cite|improve this question















share|cite|improve this question













share|cite|improve this question




share|cite|improve this question








edited Jan 2 at 18:23









hardmath

29.1k953101




29.1k953101










asked Jul 15 '18 at 23:54









Ethan BolkerEthan Bolker

44.7k553120




44.7k553120




marked as duplicate by hardmath, user91500, Lord_Farin, Cesareo, Jyrki Lahtonen Jan 17 at 15:15


This question has been asked before and already has an answer. If those answers do not fully address your question, please ask a new question.









marked as duplicate by hardmath, user91500, Lord_Farin, Cesareo, Jyrki Lahtonen Jan 17 at 15:15


This question has been asked before and already has an answer. If those answers do not fully address your question, please ask a new question.










  • 1




    $begingroup$
    Why don't you cite the reference that very source that Wikipedia used: G. H. Hardy and E. M. Wright, An Introduction to the Theory of Numbers, 6th ed., Oxford University Press, 2008, p. 494.
    $endgroup$
    – fleablood
    Jul 16 '18 at 0:00










  • $begingroup$
    @fleablood Of course. I missed that reference. Post as an answer, or I can delete the question as too narrow.
    $endgroup$
    – Ethan Bolker
    Jul 16 '18 at 0:04














  • 1




    $begingroup$
    Why don't you cite the reference that very source that Wikipedia used: G. H. Hardy and E. M. Wright, An Introduction to the Theory of Numbers, 6th ed., Oxford University Press, 2008, p. 494.
    $endgroup$
    – fleablood
    Jul 16 '18 at 0:00










  • $begingroup$
    @fleablood Of course. I missed that reference. Post as an answer, or I can delete the question as too narrow.
    $endgroup$
    – Ethan Bolker
    Jul 16 '18 at 0:04








1




1




$begingroup$
Why don't you cite the reference that very source that Wikipedia used: G. H. Hardy and E. M. Wright, An Introduction to the Theory of Numbers, 6th ed., Oxford University Press, 2008, p. 494.
$endgroup$
– fleablood
Jul 16 '18 at 0:00




$begingroup$
Why don't you cite the reference that very source that Wikipedia used: G. H. Hardy and E. M. Wright, An Introduction to the Theory of Numbers, 6th ed., Oxford University Press, 2008, p. 494.
$endgroup$
– fleablood
Jul 16 '18 at 0:00












$begingroup$
@fleablood Of course. I missed that reference. Post as an answer, or I can delete the question as too narrow.
$endgroup$
– Ethan Bolker
Jul 16 '18 at 0:04




$begingroup$
@fleablood Of course. I missed that reference. Post as an answer, or I can delete the question as too narrow.
$endgroup$
– Ethan Bolker
Jul 16 '18 at 0:04










2 Answers
2






active

oldest

votes


















3












$begingroup$

The ariticle on wikipededia cites that with a foot note (as of the time I write this) #8 refering to: G. H. Hardy and E. M. Wright, An Introduction to the Theory of Numbers, 6th ed., Oxford University Press, 2008, p. 494.




from Wikipedia



It follows from the prime number theorem that for any real $ {displaystyle varepsilon >0}$ there is a ${displaystyle n_{0}>0} $ such that for all ${displaystyle n>n_{0}} $ there is a prime $ {displaystyle p}$ such ${displaystyle n<p<(1+varepsilon )n}$. It can be shown, for instance,
that



${displaystyle lim _{nto infty }{frac {pi ((1+varepsilon )n)-pi
> (n)}{n/log n}}=varepsilon ,} $[8]



[8]G. H. Hardy and E. M. Wright, An Introduction to the Theory of Numbers, 6th ed., Oxford University Press, 2008, p. 494.







share|cite|improve this answer











$endgroup$













  • $begingroup$
    This is pretty unreadable. I doubt that Hardy and Wright says “for any real $epsilon > 0 > epsilon > 0epsilon > 0$, for example (and it doesn’t get better). Can you edit the quote so it doesn’t look like a copy/paste from some non-LaTeX source? Even an image of the relevant Hardy and Wright page would be better than this.
    $endgroup$
    – Steve Kass
    Jul 16 '18 at 0:34












  • $begingroup$
    It is a copy and paste from a non-LaTeX source! I thought the was clear from context.
    $endgroup$
    – fleablood
    Jul 16 '18 at 2:19



















2












$begingroup$

tersely worded, but Dusart gives, for $x geq 396738,$ a prime between $x$ and
$$ x left( 1 + frac{1}{25 log^2 x} right) $$



I think it was on the arxiv, let me find it.



Yes, this is Proposition 6.8.



For your fixed target $3x/2,$ you may fill in the smaller numbers with the table of maximal prime gaps at https://en.wikipedia.org/wiki/Prime_gap#Numerical_results . I went through the thing once by machine, for $p geq 11$ and $p < 4 cdot 10^{18},$ we get gap $g < log^2 p.$






share|cite|improve this answer











$endgroup$




















    2 Answers
    2






    active

    oldest

    votes








    2 Answers
    2






    active

    oldest

    votes









    active

    oldest

    votes






    active

    oldest

    votes









    3












    $begingroup$

    The ariticle on wikipededia cites that with a foot note (as of the time I write this) #8 refering to: G. H. Hardy and E. M. Wright, An Introduction to the Theory of Numbers, 6th ed., Oxford University Press, 2008, p. 494.




    from Wikipedia



    It follows from the prime number theorem that for any real $ {displaystyle varepsilon >0}$ there is a ${displaystyle n_{0}>0} $ such that for all ${displaystyle n>n_{0}} $ there is a prime $ {displaystyle p}$ such ${displaystyle n<p<(1+varepsilon )n}$. It can be shown, for instance,
    that



    ${displaystyle lim _{nto infty }{frac {pi ((1+varepsilon )n)-pi
    > (n)}{n/log n}}=varepsilon ,} $[8]



    [8]G. H. Hardy and E. M. Wright, An Introduction to the Theory of Numbers, 6th ed., Oxford University Press, 2008, p. 494.







    share|cite|improve this answer











    $endgroup$













    • $begingroup$
      This is pretty unreadable. I doubt that Hardy and Wright says “for any real $epsilon > 0 > epsilon > 0epsilon > 0$, for example (and it doesn’t get better). Can you edit the quote so it doesn’t look like a copy/paste from some non-LaTeX source? Even an image of the relevant Hardy and Wright page would be better than this.
      $endgroup$
      – Steve Kass
      Jul 16 '18 at 0:34












    • $begingroup$
      It is a copy and paste from a non-LaTeX source! I thought the was clear from context.
      $endgroup$
      – fleablood
      Jul 16 '18 at 2:19
















    3












    $begingroup$

    The ariticle on wikipededia cites that with a foot note (as of the time I write this) #8 refering to: G. H. Hardy and E. M. Wright, An Introduction to the Theory of Numbers, 6th ed., Oxford University Press, 2008, p. 494.




    from Wikipedia



    It follows from the prime number theorem that for any real $ {displaystyle varepsilon >0}$ there is a ${displaystyle n_{0}>0} $ such that for all ${displaystyle n>n_{0}} $ there is a prime $ {displaystyle p}$ such ${displaystyle n<p<(1+varepsilon )n}$. It can be shown, for instance,
    that



    ${displaystyle lim _{nto infty }{frac {pi ((1+varepsilon )n)-pi
    > (n)}{n/log n}}=varepsilon ,} $[8]



    [8]G. H. Hardy and E. M. Wright, An Introduction to the Theory of Numbers, 6th ed., Oxford University Press, 2008, p. 494.







    share|cite|improve this answer











    $endgroup$













    • $begingroup$
      This is pretty unreadable. I doubt that Hardy and Wright says “for any real $epsilon > 0 > epsilon > 0epsilon > 0$, for example (and it doesn’t get better). Can you edit the quote so it doesn’t look like a copy/paste from some non-LaTeX source? Even an image of the relevant Hardy and Wright page would be better than this.
      $endgroup$
      – Steve Kass
      Jul 16 '18 at 0:34












    • $begingroup$
      It is a copy and paste from a non-LaTeX source! I thought the was clear from context.
      $endgroup$
      – fleablood
      Jul 16 '18 at 2:19














    3












    3








    3





    $begingroup$

    The ariticle on wikipededia cites that with a foot note (as of the time I write this) #8 refering to: G. H. Hardy and E. M. Wright, An Introduction to the Theory of Numbers, 6th ed., Oxford University Press, 2008, p. 494.




    from Wikipedia



    It follows from the prime number theorem that for any real $ {displaystyle varepsilon >0}$ there is a ${displaystyle n_{0}>0} $ such that for all ${displaystyle n>n_{0}} $ there is a prime $ {displaystyle p}$ such ${displaystyle n<p<(1+varepsilon )n}$. It can be shown, for instance,
    that



    ${displaystyle lim _{nto infty }{frac {pi ((1+varepsilon )n)-pi
    > (n)}{n/log n}}=varepsilon ,} $[8]



    [8]G. H. Hardy and E. M. Wright, An Introduction to the Theory of Numbers, 6th ed., Oxford University Press, 2008, p. 494.







    share|cite|improve this answer











    $endgroup$



    The ariticle on wikipededia cites that with a foot note (as of the time I write this) #8 refering to: G. H. Hardy and E. M. Wright, An Introduction to the Theory of Numbers, 6th ed., Oxford University Press, 2008, p. 494.




    from Wikipedia



    It follows from the prime number theorem that for any real $ {displaystyle varepsilon >0}$ there is a ${displaystyle n_{0}>0} $ such that for all ${displaystyle n>n_{0}} $ there is a prime $ {displaystyle p}$ such ${displaystyle n<p<(1+varepsilon )n}$. It can be shown, for instance,
    that



    ${displaystyle lim _{nto infty }{frac {pi ((1+varepsilon )n)-pi
    > (n)}{n/log n}}=varepsilon ,} $[8]



    [8]G. H. Hardy and E. M. Wright, An Introduction to the Theory of Numbers, 6th ed., Oxford University Press, 2008, p. 494.








    share|cite|improve this answer














    share|cite|improve this answer



    share|cite|improve this answer








    edited Jul 16 '18 at 2:28

























    answered Jul 16 '18 at 0:12









    fleabloodfleablood

    71.9k22687




    71.9k22687












    • $begingroup$
      This is pretty unreadable. I doubt that Hardy and Wright says “for any real $epsilon > 0 > epsilon > 0epsilon > 0$, for example (and it doesn’t get better). Can you edit the quote so it doesn’t look like a copy/paste from some non-LaTeX source? Even an image of the relevant Hardy and Wright page would be better than this.
      $endgroup$
      – Steve Kass
      Jul 16 '18 at 0:34












    • $begingroup$
      It is a copy and paste from a non-LaTeX source! I thought the was clear from context.
      $endgroup$
      – fleablood
      Jul 16 '18 at 2:19


















    • $begingroup$
      This is pretty unreadable. I doubt that Hardy and Wright says “for any real $epsilon > 0 > epsilon > 0epsilon > 0$, for example (and it doesn’t get better). Can you edit the quote so it doesn’t look like a copy/paste from some non-LaTeX source? Even an image of the relevant Hardy and Wright page would be better than this.
      $endgroup$
      – Steve Kass
      Jul 16 '18 at 0:34












    • $begingroup$
      It is a copy and paste from a non-LaTeX source! I thought the was clear from context.
      $endgroup$
      – fleablood
      Jul 16 '18 at 2:19
















    $begingroup$
    This is pretty unreadable. I doubt that Hardy and Wright says “for any real $epsilon > 0 > epsilon > 0epsilon > 0$, for example (and it doesn’t get better). Can you edit the quote so it doesn’t look like a copy/paste from some non-LaTeX source? Even an image of the relevant Hardy and Wright page would be better than this.
    $endgroup$
    – Steve Kass
    Jul 16 '18 at 0:34






    $begingroup$
    This is pretty unreadable. I doubt that Hardy and Wright says “for any real $epsilon > 0 > epsilon > 0epsilon > 0$, for example (and it doesn’t get better). Can you edit the quote so it doesn’t look like a copy/paste from some non-LaTeX source? Even an image of the relevant Hardy and Wright page would be better than this.
    $endgroup$
    – Steve Kass
    Jul 16 '18 at 0:34














    $begingroup$
    It is a copy and paste from a non-LaTeX source! I thought the was clear from context.
    $endgroup$
    – fleablood
    Jul 16 '18 at 2:19




    $begingroup$
    It is a copy and paste from a non-LaTeX source! I thought the was clear from context.
    $endgroup$
    – fleablood
    Jul 16 '18 at 2:19











    2












    $begingroup$

    tersely worded, but Dusart gives, for $x geq 396738,$ a prime between $x$ and
    $$ x left( 1 + frac{1}{25 log^2 x} right) $$



    I think it was on the arxiv, let me find it.



    Yes, this is Proposition 6.8.



    For your fixed target $3x/2,$ you may fill in the smaller numbers with the table of maximal prime gaps at https://en.wikipedia.org/wiki/Prime_gap#Numerical_results . I went through the thing once by machine, for $p geq 11$ and $p < 4 cdot 10^{18},$ we get gap $g < log^2 p.$






    share|cite|improve this answer











    $endgroup$


















      2












      $begingroup$

      tersely worded, but Dusart gives, for $x geq 396738,$ a prime between $x$ and
      $$ x left( 1 + frac{1}{25 log^2 x} right) $$



      I think it was on the arxiv, let me find it.



      Yes, this is Proposition 6.8.



      For your fixed target $3x/2,$ you may fill in the smaller numbers with the table of maximal prime gaps at https://en.wikipedia.org/wiki/Prime_gap#Numerical_results . I went through the thing once by machine, for $p geq 11$ and $p < 4 cdot 10^{18},$ we get gap $g < log^2 p.$






      share|cite|improve this answer











      $endgroup$
















        2












        2








        2





        $begingroup$

        tersely worded, but Dusart gives, for $x geq 396738,$ a prime between $x$ and
        $$ x left( 1 + frac{1}{25 log^2 x} right) $$



        I think it was on the arxiv, let me find it.



        Yes, this is Proposition 6.8.



        For your fixed target $3x/2,$ you may fill in the smaller numbers with the table of maximal prime gaps at https://en.wikipedia.org/wiki/Prime_gap#Numerical_results . I went through the thing once by machine, for $p geq 11$ and $p < 4 cdot 10^{18},$ we get gap $g < log^2 p.$






        share|cite|improve this answer











        $endgroup$



        tersely worded, but Dusart gives, for $x geq 396738,$ a prime between $x$ and
        $$ x left( 1 + frac{1}{25 log^2 x} right) $$



        I think it was on the arxiv, let me find it.



        Yes, this is Proposition 6.8.



        For your fixed target $3x/2,$ you may fill in the smaller numbers with the table of maximal prime gaps at https://en.wikipedia.org/wiki/Prime_gap#Numerical_results . I went through the thing once by machine, for $p geq 11$ and $p < 4 cdot 10^{18},$ we get gap $g < log^2 p.$







        share|cite|improve this answer














        share|cite|improve this answer



        share|cite|improve this answer








        edited Jul 16 '18 at 0:38

























        answered Jul 16 '18 at 0:22









        Will JagyWill Jagy

        104k5102201




        104k5102201















            Popular posts from this blog

            Cabo Verde

            Karlovacs län

            Gyllenstierna